Đến nội dung

Hình ảnh

Topic BẤT ĐẲNG THỨC ôn thi vào lớp 10 THPT 2017 - 2018

bất đẳng thức am-gm cauchy bunyakovski minskovski schwarz holder thcs

  • Please log in to reply
Chủ đề này có 299 trả lời

#21
Minhnksc

Minhnksc

    Sĩ quan

  • Điều hành viên OLYMPIC
  • 302 Bài viết

 

 

Bài 11 : (Thi thử KHTN đợt 3 vòng 2)

Cho các số $a,b,c$ thỏa mãn $0\leq a,b,c \leq 2$ và $a+b+c=3$. Chứng minh rằng

$a^{2}+b^{2}+c^{2}\leq 5$

 

 

Bài 11: Giả sử a=max{a;b;c}
Mà $a+b+c=3$ nên $a\geq 1$
Vì $a=3-b-c$, do đó bất đẳng thức cần chứng minh tương đương:
$b^2+c^2+bc-3b-3c+2\leq 0$
Ta có $b^2+c^2+bc-3b-3c+2\leq b^2+c^2+2bc-3b-3c+2 =(b+c-1)(b+c-2)$(1)
Lại có $b+c=3-a$ và $1\leq a \leq 2$  nên $1\leq b+c \leq 2$
Kết hợp với (1) suy ra đpcm.

Sống khỏe và sống tốt :D


#22
Minhnksc

Minhnksc

    Sĩ quan

  • Điều hành viên OLYMPIC
  • 302 Bài viết

Bài 14:(thi thử lần 4 KHTN 2014-2015)

Cho a;b;c là các số thực dương thỏa $abc=a+b+c+2$, chứng minh rằng

$\sum\sqrt{\frac{1}{a+1}}\leq \sqrt{3}$


Sống khỏe và sống tốt :D


#23
Nguyenphuctang

Nguyenphuctang

    Sĩ quan

  • Banned
  • 499 Bài viết

Lời giải khác cho bài 11: 

 

Vì $a,b,c\in [0;2]\Rightarrow (2-a)(2-b)(2-c)\geq 0\Leftrightarrow \sum ab \geq 2+ \frac{abc}{2}\geq 2$

 

Bất đẳng thức cần chứng minh tương đương với: 

 

$9- 2\sum ab \leq 5 \Leftrightarrow  \sum ab \geq 2$

 

Bài toán được chứng minh xong.

 

Đẳng thức xảy ra khi và chỉ khi:$ a=2 ; b= 1; c= 0$ và các cặp hoán vị.
Các bạn giải nốt các bài còn lại tối mình post tiếp. Topic vừa mở chưa đầy 24 giờ mà đã có 24 post là rất mừng rồi. Cám ơn các bạn ủng hộ


Bài viết đã được chỉnh sửa nội dung bởi Nguyenphuctang: 18-04-2017 - 17:39


#24
NHoang1608

NHoang1608

    Sĩ quan

  • Thành viên
  • 375 Bài viết

Bài 14:(thi thử lần 4 KHTN 2014-2015)

Cho a;b;c là các số thực dương thỏa $abc=a+b+c+2$, chứng minh rằng

$\sum\sqrt{\frac{1}{a+1}}\leq \sqrt{3}$

$\boxed{\text{Lời giải bài 14}}$

Theo GT thì ta có thể đặt $(a;b;c)=(\frac{x+y}{z};\frac{y+z}{x};\frac{z+x}{y})$ với $x,y,z > 0$

Khi đó $\sum\sqrt{\frac{1}{a+1}} = \sum\sqrt{\frac{z}{x+y+z}}$

Mặt khác thì $(\sum\sqrt{\frac{z}{x+y+z}})^{2} \leq 3\sum\frac{z}{x+y+z} =3 $

                     $\Rightarrow \sum\sqrt{\frac{z}{x+y+z}} \leq \sqrt{3}$  

Hay $\sum\sqrt{\frac{1}{a+1}}\leq \sqrt{3}$

Đẳng thức xảy ra khi $a=b=c=2$


Bài viết đã được chỉnh sửa nội dung bởi NHoang1608: 18-04-2017 - 18:27

The greatest danger for most of us is not that our aim is too high and we miss it, but that it is too low and we reach it.

----- Michelangelo----


#25
tuaneee111

tuaneee111

    Trung sĩ

  • Thành viên
  • 174 Bài viết

 

$\sum \frac{a^2}{b^2+c^2}$ $\geq$ $\sum\frac{a}{b+c}$  (Nâng cao pt 8 tập 2)
$\sum \frac{a}{b+c}$  $\geq$  $\frac{(a+b+c)^2)}{2(ab+bc+ca)}$  (C-S)

 

Theo bđt Cauchy - Schwarz ta có:

\[\sum\limits_{cyc} {\frac{{{a^2}}}{{{b^2} + {c^2}}}}  - \frac{{{{\left( {a + b + c} \right)}^2}}}{{2\left( {ab + bc + ca} \right)}} \ge \frac{{{{\left( {{a^2} + {b^2} + {c^2}} \right)}^2}}}{{2\left( {{a^2}{b^2} + {b^2}{c^2} + {c^2}{a^2}} \right)}} - \frac{{{{\left( {a + b + c} \right)}^2}}}{{2\left( {ab + bc + ca} \right)}}\]

Đặt ab+ba+ca=1 Đổi biến về pqr ta cần chứng minh:

\[f\left( p \right) = {p^4} - 5{p^2} + 2{p^3}r + 4 \ge 0\]

Nếu p>2 thì bđt đúng

Nếu \[p \in \left[ {\sqrt 3 ;2} \right]\] thì theo Schur ta có:

\[f\left( p \right) \ge {p^4} - 5{p^2} + \frac{{2{p^4}\left( {4 - {p^2}} \right)}}{9} + 4 = \frac{{\left( {{p^2} - 3} \right)\left( { - 2{p^4} + 11{p^2} - 12} \right)}}{9} \ge 0\]

Bất đẳng thức cuối đúng nên có đpcm


Bài viết đã được chỉnh sửa nội dung bởi tuaneee111: 18-04-2017 - 20:13

$$\boxed{\boxed{I\heartsuit MATHEMATICAL}}$$

Blog của tôi

:luoi: Sức hấp dẫn của toán học mãnh liệt đến nỗi tôi bắt đầu sao nhãng các môn học khác - Sofia Vasilyevna Kovalevskaya :lol:


#26
cyndaquil

cyndaquil

    Hạ sĩ

  • Thành viên
  • 63 Bài viết

Ủng hộ

 

Bài 14:(thi thử lần 4 KHTN 2014-2015)

Cho a;b;c là các số thực dương thỏa $abc=a+b+c+2$, chứng minh rằng

$\sum\sqrt{\frac{1}{a+1}}\leq \sqrt{3}$

 Ta có $a+b+c+2=abc \Leftrightarrow \frac 1{a+1}+\frac 1{b+1}+\frac 1{c+1}=1$

Nên $VT \le \sqrt{3\sum \frac 1{a+1}}=\sqrt 3$


Bài viết đã được chỉnh sửa nội dung bởi cyndaquil: 18-04-2017 - 20:13


#27
tuaneee111

tuaneee111

    Trung sĩ

  • Thành viên
  • 174 Bài viết

Mình xin nhắc thêm là mình sẽ ít đăng những bài bất cũ và quá quen. Thay vào đó sẽ đăng những bài toán mới sáng tác,..... chủ yếu là rèn luyện tư duy tránh tư tưởng ''nhớ lại''. Vì hiện nay xu hướng là sáng tác đề không cho lại các bài toán cũ.

Bài toán đề xuất tiếp theo:

 

Bài 4: (Sưu tầm)

Cho a, b, c là các số thực không âm thỏa mãn không có 2 số nào đồng thời bằng không. Chứng minh rằng:

$\sum \frac{a^{2}}{b^{2}+c^{2}} \geq \frac{(a+b+c)^{2}}{2(ab+bc+ca)}$

 

Bài 5: (Olympic 30/4 2017)

Cho a, b, c > 0. Chứng minh rằng:

$\sum \frac{a^{4}+27}{b+c} \geq 2(a^{2}+b^{2}+c{^2})$

 

Bài 6: (Nguyễn Việt Hùng, HSGS)

Cho a, b, c > 0. Chứng minh rằng:

$\sum \frac{a^{2}}{b+c} \geq \frac{a^{3}+b^{3}+c^{3}}{6(ab+bc+ca)}+\frac{4}{9}(a+b+c)$

Bài 5.

Ta có:

\[\frac{{{a^4} + 27}}{{b + c}} + 3\left( {b + c} \right) \ge 2\sqrt {3\left( {{a^4} + 27} \right)} \]

Do đó:

\[\sum\limits_{cyc} {\frac{{{a^4} + 27}}{{b + c}}}  - 2\sum\limits_{cyc} {{a^2}}  \ge \sum\limits_{cyc} {\left( {2\sqrt {3\left( {{a^4} + 27} \right)}  - 6a - 2{a^2}} \right)}  = \sum\limits_{cyc} {\frac{{{{\left( {a - 3} \right)}^2}\left( {8{a^2} + 24a + 36} \right)}}{{2\sqrt {3\left( {{a^4} + 27} \right)}  + 2{a^2} + 6a}}}  \ge 0\]


Bài viết đã được chỉnh sửa nội dung bởi tuaneee111: 19-04-2017 - 07:12

$$\boxed{\boxed{I\heartsuit MATHEMATICAL}}$$

Blog của tôi

:luoi: Sức hấp dẫn của toán học mãnh liệt đến nỗi tôi bắt đầu sao nhãng các môn học khác - Sofia Vasilyevna Kovalevskaya :lol:


#28
viet9a14124869

viet9a14124869

    Trung úy

  • Thành viên
  • 903 Bài viết

Bài 5.

Không mất tính tq ta gs: \[{a^2} + {b^2} + {c^2} = 3\]

Ta có:

\[\sum\limits_{cyc} {\frac{{{a^4} + 27}}{{b + c}}}  \ge \sum\limits_{cyc} {\frac{{{a^4} + 27}}{{\sqrt {2\left( {{b^2} + {c^2}} \right)} }} = \sum\limits_{cyc} {\frac{{{a^4} + 27}}{{\sqrt {2\left( {3 - {a^2}} \right)} }}} } \]

Ta chứng minh đánh giá sau đây đúng:

\[\frac{{{a^4} + 27}}{{\sqrt {2\left( {3 - {a^2}} \right)} }} \ge \frac{9}{2}{a^2} + \frac{{19}}{2} \Leftrightarrow {\left( {a - 1} \right)^2}\left( {4{a^6} + 8{a^5} + 174{a^4} + 340{a^3} + 920{a^2} + 1500a + 750} \right) \ge 0\]

Đánh giá cuối đúng nên có đpcm!

( hình như đề bài có vấn đề)

Bài toán không thuộc dạng chuẩn hóa do chỉ có dấu bằng duy nhất là a=b=c=3

Lời giải bài 5 này mình đã đăng ở trang 1 ,bạn xem lại nhé !


                                                                    SÓNG BẮT ĐẦU TỪ GIÓ

                                                                    GIÓ BẮT ĐẦU TỪ ĐÂU ?

                                                                    ANH CŨNG KHÔNG BIẾT NỮA 

                                                                    KHI NÀO...? TA YÊU NHAU .


#29
Nguyen Xuan Hieu

Nguyen Xuan Hieu

    Binh nhất

  • Thành viên mới
  • 39 Bài viết

Bài 15:(Nguyen Xuan Hieu)
Cho $a,b,c>0$.Chứng minh rằng:
$7(a^3+b^3+c^3)+6 \geq 3\sqrt{ac}(2ac+1)+3\sqrt{bc}(2bc+1)+3\sqrt{ab}(2ab+1)$
 



#30
sharker

sharker

    Sĩ quan

  • Thành viên
  • 301 Bài viết

 

Bài 10 : (T6 Tạp chí THTT số 474) 

Với $x$ là số thực thay đổi tìm giá trị nhỏ nhất của biểu thức:

f= $\sqrt{x^{2}-2x+2} + \sqrt{x^{2}-8x+32}+ \sqrt{x^{2}-6x+25} + \sqrt{x^{2}-4x+20}+ \sqrt{x^{2}-10x+26}$

 

 
$f(x) = \left( {\sqrt {{{(1 - x)}^2} + 1}  + \sqrt {{{(x - 5)}^2} + 1} } \right) + \left( {\sqrt {{{(x - 4)}^2} + 16}  + \sqrt {{{(2 - x)}^2} + 16} } \right) + \sqrt {{{(x - 3)}^2} + 16}$
 $\ge \sqrt {{4^2} + {2^2}}  + \sqrt {{2^2} + {8^2}}  + \sqrt {16}  = \sqrt {20}  + \sqrt {68}  + 4$
Vậy min f(x)= $\sqrt {20}  + \sqrt {68}  + 4$ tại $x=3$

Anh sẽ vẫn bên em dù bất cứ nơi đâu

Anh sẽ là hạt bụi bay theo gió

Anh sẽ là ngôi sao trên bầu trời phương Bắc

Anh không bao giờ dừng lại ở một nơi nào

Anh sẽ là ngọn gió thổi qua các ngọn cây

Em sẽ mãi mãi đợi anh chứ ??

will you wait for me forever


#31
Nghiapnh1002

Nghiapnh1002

    Trung sĩ

  • Thành viên
  • 108 Bài viết

Bài 16: Cho các số thực dương thỏa mãn $a+b+c+\sqrt{2abc} \geq 10$

Chứng minh rằng:

$\sum \sqrt{\frac{8}{a^2}+\frac{9b^2}{2}+\frac{c^2a^2}{4}}  \geq 6\sqrt{6}$


Bài viết đã được chỉnh sửa nội dung bởi Nghiapnh1002: 19-04-2017 - 13:30


#32
Mr Cooper

Mr Cooper

    Sĩ quan

  • Thành viên
  • 496 Bài viết

$\boxed{8}$ [Trần Quốc Anh] Cho $a,b,c$ là các số thực không âm thỏa mãn $a+b+c=2$. Chứng minh rằng:

\[(a^2+ab+b^2)(b^2+bc+c^2)(c^2+ca+a^2) \le 3 \]

$\boxed{8}$

SOS

\[RHS-LHS=\frac{1}{2}\sum{a(a^2b+a^2c+2b+2c+2)(a-1)^2}+(a-1)^2(b-1)^2(c-1)^2+\frac{1}{2}a^2b^2c^2\ge{0}\]


Bài viết đã được chỉnh sửa nội dung bởi Mr Cooper: 18-04-2017 - 22:29


#33
Minhnksc

Minhnksc

    Sĩ quan

  • Điều hành viên OLYMPIC
  • 302 Bài viết

Bài 15:(Nguyen Xuan Hieu)
Cho $a,b,c>0$.Chứng minh rằng:
$7(a^3+b^3+c^3)+6 \geq 3\sqrt{ac}(2ac+1)+3\sqrt{bc}(2bc+1)+3\sqrt{ab}(2ab+1)$
 

áp dụng BĐT AM-GM, ta có

$3\sum_{cyc}(a^3+b^3)\geq 6\sum_{cyc} ab\sqrt{ab}$

$a^3+b^3+c^3+6=\frac{1}{2}\sum_{cyc}(a^3+1+1)+(b^3+1+1)\geq 3(a+b+c)\geq 3(\sqrt{ab}+\sqrt{bc}+\sqrt{ac})$

cộng các vế ta có đpcm.

Dấu bằng xảy ra khi $a=b=c=1$.


Bài viết đã được chỉnh sửa nội dung bởi Minhnksc: 18-04-2017 - 22:37

Sống khỏe và sống tốt :D


#34
Nguyenhuyen_AG

Nguyenhuyen_AG

    Trung úy

  • Thành viên nổi bật 2016
  • 945 Bài viết

$\boxed{8}$ [Trần Quốc Anh] Cho $a,b,c$ là các số thực không âm thỏa mãn $a+b+c=2$. Chứng minh rằng:

\[(a^2+ab+b^2)(b^2+bc+c^2)(c^2+ca+a^2) \le 3 \]

 

Bất đẳng thức cần chứng minh tương đương với

\[\frac{3}{64}(a+b+c)^6 \geqslant (a^2+ab+b^2)(b^2+bc+c^2)(c^2+ca+a^2),\]

hoặc

\[\frac{5}{16}(a+b+c)^2 \sum ab(a-b)^2+\frac{1}{64}\left(\sum a^2-2\sum bc\right)^2\left(3\sum a^2 + 10 \sum bc \right) + \frac{1}{16}abc(a+b+c)^3 \geqslant 0.\]

Từ đó suy điều phải chứng minh.

 

P/s. Bài này không phải của Trần Quốc Anh đâu.


Nguyen Van Huyen
Ho Chi Minh City University Of Transport

#35
Nguyen Xuan Hieu

Nguyen Xuan Hieu

    Binh nhất

  • Thành viên mới
  • 39 Bài viết

Bài 17:(Nguyen Xuan Hieu)
Cho $a,b,c$ là các số thực dương thõa mãn  $a+b+c=3$.Chứng minh rằng:
$\sum_{cyc} \dfrac{a^2+1}{3\sqrt[3]{a^2b^2}(1+3\sqrt[3]{c^2})-8} \geq \dfrac{3}{2}$
 


Bài viết đã được chỉnh sửa nội dung bởi Nguyen Xuan Hieu: 20-04-2017 - 04:55


#36
tuaneee111

tuaneee111

    Trung sĩ

  • Thành viên
  • 174 Bài viết

Mình xin nhắc thêm là mình sẽ ít đăng những bài bất cũ và quá quen. Thay vào đó sẽ đăng những bài toán mới sáng tác,..... chủ yếu là rèn luyện tư duy tránh tư tưởng ''nhớ lại''. Vì hiện nay xu hướng là sáng tác đề không cho lại các bài toán cũ.

Bài toán đề xuất tiếp theo:

 

Bài 4: (Sưu tầm)

Cho a, b, c là các số thực không âm thỏa mãn không có 2 số nào đồng thời bằng không. Chứng minh rằng:

$\sum \frac{a^{2}}{b^{2}+c^{2}} \geq \frac{(a+b+c)^{2}}{2(ab+bc+ca)}$

 

Bài 5: (Olympic 30/4 2017)

Cho a, b, c > 0. Chứng minh rằng:

$\sum \frac{a^{4}+27}{b+c} \geq 2(a^{2}+b^{2}+c{^2})$

 

Bài 6: (Nguyễn Việt Hùng, HSGS)

Cho a, b, c > 0. Chứng minh rằng:

$\sum \frac{a^{2}}{b+c} \geq \frac{a^{3}+b^{3}+c^{3}}{6(ab+bc+ca)}+\frac{4}{9}(a+b+c)$

Bài 6:

Giả sử \[c = \min \left\{ {a,b,c} \right\}\]

Ta có:

\[\sum\limits_{cyc} {\frac{{{a^2}}}{{b + c}}}  - \frac{4}{9}\left( {a + b + c} \right) - \frac{{{a^3} + {b^3} + {c^3}}}{{6\left( {ab + bc + ca} \right)}} = {\left( {a - b} \right)^2}\left( {\underbrace {\frac{{a + b + c}}{{\left( {a + c} \right)\left( {b + c} \right)}} - \frac{{3a + 3b + c}}{{18\left( {ab + bc + ca} \right)}}}_P} \right) + \left( {a - c} \right)\left( {b - c} \right)\left( {\underbrace {\frac{{\left( {a + b + c} \right)\left( {a + b + 2c} \right)}}{{2\left( {a + b} \right)\left( {b + c} \right)\left( {c + a} \right)}} - \frac{{2a + 2b + 3c}}{{18\left( {ab + bc + ca} \right)}}}_T} \right)\]

Mặt khác:

\[P > \frac{{\left( {a + b + c} \right)\left( {5\left( {ab + bc + ca} \right) - {c^2}} \right)}}{{6\left( {ab + bc + ca} \right)\left( {ab + bc + ca + {c^2}} \right)}} > 0\]

\[T > \left( {a + b + c} \right)\left( {\frac{{2\left( {a + b + c} \right)\left( {ab + bc + ca} \right) - abc}}{{6\left( {a + b} \right)\left( {b + c} \right)\left( {c + a} \right)\left( {ab + bc + ca} \right)}}} \right) > 0\]

Vậy bđt đc cm


$$\boxed{\boxed{I\heartsuit MATHEMATICAL}}$$

Blog của tôi

:luoi: Sức hấp dẫn của toán học mãnh liệt đến nỗi tôi bắt đầu sao nhãng các môn học khác - Sofia Vasilyevna Kovalevskaya :lol:


#37
TrBaoChis

TrBaoChis

    Hạ sĩ

  • Banned
  • 81 Bài viết

Bài 7 (@Phamphucduc): 
Cho $a,b,c$ là các số thực dương thỏa mãn $a^{2}+b^{2}+c^{2}\leq 3b$
Chứng minh rằng $\frac{1}{(a+1)^{2}}+\frac{4}{(b+2)^{2}}+\frac{8}{(c+3)^{2}}\geq 1$
 

Spoiler

 

lời giải ntn bạn 



#38
NHoang1608

NHoang1608

    Sĩ quan

  • Thành viên
  • 375 Bài viết

Lời giải bài 7:

                   Theo giả thiết ta có $3b\geq (\frac{b^{2}}{2})+(\frac{b^{2}}{4}+c^{2})+(\frac{b^{2}}{4}+c^{2})$

                   Mà $\frac{b^{2}}{4}+c^{2}\geq bc$ và $\frac{b^{2}}{4}+a^{2}\geq ab$

                   Suy ra $3b\geq \frac{b^{2}}{2}+bc+ab$

                   $\Rightarrow 3 \geq \frac{b}{2}+a+c$ $(1)$

 

     Ta có:

$P=\frac{1}{(a+1)^{2}}+\frac{4}{(b+2)^{2}}+\frac{4}{(c+3)^{2}}+\frac{4}{(c+3)^{2}}$

           $ \geq \frac{1}{4}(\frac{1}{a+1}+\frac{2}{b+2}+\frac{2}{c+3}+\frac{2}{c+3})^{2}$

           $=\frac{1}{4}(\frac{1}{a+1}+\frac{1}{\frac{b}{2}+1}+\frac{4}{c+3})^{2}$

           $\geq \frac{1}{4}(\frac{(1+1+2)^{2}}{a+1+\frac{b}{2}+1+c+3})^{2}$

$\Rightarrow P \geq \frac{1}{4}(\frac{(1+1+2)^{2}}{a+1+\frac{b}{2}+1+c+3})^{2}$  $(2)$

Sử dụng $(1)$ thì ta được $\frac{1}{4}(\frac{(1+1+2)^{2}}{a+1+\frac{b}{2}+1+c+3})^{2} \geq \frac{1}{4}(\frac{4^{2}}{3+5})^{2}= 1$  $(3)$

 Từ $(2)$ và $(3)$ thì ta được $P \geq 1$

Đẳng thức xảy ra khi $a=c=1, b=2$


The greatest danger for most of us is not that our aim is too high and we miss it, but that it is too low and we reach it.

----- Michelangelo----


#39
NHoang1608

NHoang1608

    Sĩ quan

  • Thành viên
  • 375 Bài viết

Bài 17:
Cho $a,b,c$ là các số thực dương thõa mãn  $a+b+c=3$.Chứng minh rằng:
$\sum_{cyc} \dfrac{a^2+1}{3\sqrt[3]{a^2b^2}(1+3\sqrt[3]{c^2})-8} \geq \dfrac{3}{2}$
 

Lời giải bài 17:

Ta có các bất đẳng thức sau:

         $3=a+b+c\geq 3\sqrt[3]{abc} \Rightarrow abc\leq 1 $

         

          $9=(a+b+c)^{2}\geq 3(ab+bc+ca) \Rightarrow ab+bc+ca\leq 3$

       

          $3\sqrt[3]{a^{2}b^{2}}(3\sqrt[3]{c^{2}}+1)= 9\sqrt[3]{a^{2}b^{2}c^{2}}+3\sqrt[3]{a^{2}b^{2}} \leq [3(a^{2}b^{2}c^{2}+1+1)]+                   (a^{2}b^{2}+1+1)$     $(AM-GM)$

 

           $\Rightarrow    3\sqrt[3]{a^{2}b^{2}}(3\sqrt[3]{c^{2}}+1)-8 \leq 3a^{2}b^{2}c^{2}+a^{2}b^{2}$

 

           $\Rightarrow \frac{a^{2}+1}{3\sqrt[3]{a^{2}b^{2}}(3\sqrt[3]{c^{2}}+1)-8} \geq \frac{a^{2}+1}{3a^{2}b^{2}c^{2}+a^{2}b^{2}}$

 

           $\Rightarrow \sum_{cyc}\frac{a^{2}+1}{3\sqrt[3]{a^{2}b^{2}}(3\sqrt[3]{c^{2}}+1)-8} \geq \sum_{cyc}\frac{a^{2}+1}         {3a^{2}b^{2}c^{2}+a^{2}b^{2}}$

 

Mà      $ \sum_{cyc}\frac{a^{2}+1}{3a^{2}b^{2}c^{2}+a^{2}b^{2}} \geq \sum_{cyc}\frac{(a+1)^{2}}{6a^{2}b^{2}c^{2}+2a^{2}b^{2}} \geq$

           $\frac{(a+b+c+1+1+1)^{2}}{18a^{2}b^{2}c^{2}+2(a^{2}b^{2}+b^{2}c^{2}+c^{2}a^{2})}= \frac{18}{   9a^{2}b^{2}c^{2}+a^{2}b^{2}+b^{2}c^{2}+c^{2}a^{2}}$  $(1)$

Mặt khác 

                       $9a^{2}b^{2}c^{2}+a^{2}b^{2}+b^{2}c^{2}+c^{2}a^{2} \leq 9abc.1 + a^{2}b^{2}+b^{2}c^{2}+c^{2}a^{2}$

                        $= 2abc(a+b+c)+ a^{2}b^{2}+b^{2}c^{2}+c^{2}a^{2} + 3abc = (ab+bc+ca)^{2}+3abc \leq 3^{2}+3= 12$

                       $\Rightarrow 9a^{2}b^{2}c^{2}+a^{2}b^{2}+b^{2}c^{2}+c^{2}a^{2} \leq 12$  $(2)$

Từ $(1)$ và $(2)$ thì $\sum_{cyc}\frac{a^{2}+1}{3a^{2}b^{2}c^{2}+a^{2}b^{2}} \geq \frac{3}{2}$

                                $\Rightarrow \sum_{cyc}\frac{a^{2}+1}{3\sqrt[3]{a^{2}b^{2}}(3\sqrt[3]{c^{2}}+1)-8} \geq \frac{3}{2}$  $(Q.E.D)$


Bài viết đã được chỉnh sửa nội dung bởi NHoang1608: 19-04-2017 - 13:02

The greatest danger for most of us is not that our aim is too high and we miss it, but that it is too low and we reach it.

----- Michelangelo----


#40
Nguyenphuctang

Nguyenphuctang

    Sĩ quan

  • Banned
  • 499 Bài viết

Các bạn chú ý lần sau đăng bài xin trích rõ nguồn nhé.

 

P/s. Bài này không phải của Trần Quốc Anh đâu.

Vâng để em nhắc Hưng sửa lại cho đúng. Đôi khi nhầm lẫn là chuyện bình thường. Bài toán đề xuất tiếp theo:

 

Bài 18: (Vasile Cirtoaje)

Cho $x, y, z > 0$ ; $x+y+z=3$. Chứng minh rằng:

$\sum_{cyc} \frac{x}{xy+1} \geq \frac{3}{2}$ 







Được gắn nhãn với một hoặc nhiều trong số những từ khóa sau: bất đẳng thức, am-gm, cauchy, bunyakovski, minskovski, schwarz, holder, thcs

0 người đang xem chủ đề

0 thành viên, 0 khách, 0 thành viên ẩn danh